Transformation of Random Variables (Z = X-Y)

Click For Summary
The discussion revolves around transforming the joint probability density function f(x,y) = e^-x * e^-y into f(z) for Z = X - Y, where X and Y are independent exponential random variables. The initial attempt involved substituting -Y with W and calculating the integral, leading to an expression that did not converge. A more effective approach was suggested, utilizing the independence of X and Y to derive the cumulative distribution function for Z, ensuring proper integration limits. The final result integrates to 1, confirming the validity of the transformation. The conversation highlights the importance of careful integration and notation in probability transformations.
Applejacks01
Messages
26
Reaction score
0

Homework Statement


Suppose we have a function, f(x,y) = e^-x * e^-y , 0<=x< ∞, 0<=y<∞,
where X and Y are exponential random variables with mean = 1. (For those who may not know, all this means is ∫(x*e^(-x) dx) from 0 to ∞ = 1, and the same for y)

Suppose we want to transform f(x,y) into f(z), where the transformation is Z = X-Y. Find f(z)


Homework Equations


f(x,y) = e^-x * e^-y , 0<=x< ∞, 0<=y<∞
Z = X-Y



The Attempt at a Solution



So I decided to transform -Y into W. So we have -Y<=W, which implies Y>=-W
∫e^-y dy from -w to ∞...
-e^-y from -w to ∞
0 + e^-(-w)
e^w

Differentiate wrt w

f(w) = e^w -∞ < w <= 0

So now we have Z = X+W
Z-W = X
We'll just let W stay as is for this problem.

The jacobian of this transformation is 1.

So we have f(z) = ∫e^-(z-w)*e^w dw from -∞ to z,

This becomes ∫e^-z*e^2w dw from -∞ to z
This becomes e^-z * (e^2w)/2 from -∞ to z
This becomes e^-z * ((e^2z)-0)/2
Which becomes (e^z)/2

The domain of z is -∞ to ∞, however, this integral does not evaluate to 1. As a matter of fact, it does not even converge.

Any help would be greatly appreciated!
 
Physics news on Phys.org
Applejacks01 said:

Homework Statement


Suppose we have a function, f(x,y) = e^-x * e^-y , 0<=x< ∞, 0<=y<∞,
where X and Y are exponential random variables with mean = 1. (For those who may not know, all this means is ∫(x*e^(-x) dx) from 0 to ∞ = 1, and the same for y)

Suppose we want to transform f(x,y) into f(z), where the transformation is Z = X-Y. Find f(z)


Homework Equations


f(x,y) = e^-x * e^-y , 0<=x< ∞, 0<=y<∞
Z = X-Y



The Attempt at a Solution



So I decided to transform -Y into W. So we have -Y<=W, which implies Y>=-W
∫e^-y dy from -w to ∞...
-e^-y from -w to ∞
0 + e^-(-w)
e^w

Differentiate wrt w

f(w) = e^w -∞ < w <= 0

So now we have Z = X+W
Z-W = X
We'll just let W stay as is for this problem.

The jacobian of this transformation is 1.

So we have f(z) = ∫e^-(z-w)*e^w dw from -∞ to z,

This becomes ∫e^-z*e^2w dw from -∞ to z
This becomes e^-z * (e^2w)/2 from -∞ to z
This becomes e^-z * ((e^2z)-0)/2
Which becomes (e^z)/2

The domain of z is -∞ to ∞, however, this integral does not evaluate to 1. As a matter of fact, it does not even converge.

Any help would be greatly appreciated!

Let f be the marginal density of X or Y. P{Z <= z} = int_{y=0..infinity} f(y)*P{X-Y <= z|Y=y} dy, and P{X-Y <= z|Y=y} = P{X <= y+z|Y=y} = P{X <= y+z} because X and Y are independent. You can get P{X <= y+z} and so do the integral. The final result is perfectly fine for all z in R and does, indeed, integrate to 1. However, you need to be careful about integration limits, since f(y) = exp(-y)*u(y) and F(y+z) = [1-exp(-y-z)]*u(y+z), where u(.) is the unit step function [u(t) = 0 for t < 0 and u(t) = 1 for t > 0]. BTW: please either use brackets (so write e^(2z)) or the "[ S U P ]" button (so write e2z) or else write "exp", as I have done. That way there is no chance of misreading what you type. Alternatively, you could use LaTeX.

RGV
 
So sorry, I saw your solution a while ago and it really helped me. I just remembered that I never thanked you/gave you credit for your solution. How do I give you credit for helping me?
 
Question: A clock's minute hand has length 4 and its hour hand has length 3. What is the distance between the tips at the moment when it is increasing most rapidly?(Putnam Exam Question) Answer: Making assumption that both the hands moves at constant angular velocities, the answer is ## \sqrt{7} .## But don't you think this assumption is somewhat doubtful and wrong?

Similar threads

  • · Replies 18 ·
Replies
18
Views
3K
Replies
11
Views
2K
Replies
8
Views
2K
Replies
2
Views
1K
Replies
7
Views
2K
Replies
2
Views
2K
Replies
6
Views
1K
Replies
2
Views
2K
  • · Replies 2 ·
Replies
2
Views
2K